skapur777
Thanks Received: 6
Atticus Finch
Atticus Finch
 
Posts: 145
Joined: March 27th, 2011
 
 
 

Q4 - A group of 1,000 students

by skapur777 Tue Apr 05, 2011 11:49 pm

Why couldn't the answer here be A? They fail to justify that 89 percent is an overwhelming majority of students because they selected a group of 1,000 kids in a medium sized city and compared it to the NATIONAL amount of students...they didn't justify this at all.
User avatar
 
bbirdwell
Thanks Received: 864
Atticus Finch
Atticus Finch
 
Posts: 803
Joined: April 16th, 2009
 
 
 

Re: Q4 - A group of 1,000 students

by bbirdwell Thu Apr 07, 2011 8:19 pm

You've pointed out the correct flaw -- that 89% of THESE students is not necessarily 89% of the NATIONAL trend. That's what (E) says, which is why (E) is correct.

(A) says something else, and it doesn't go nearly as far. It simply says "the author fails to justify that 89% is an overwhelming majority." Note that this has nothing to do with national vs local or high school students or any of that. It's much simpler. This little fact (89% = majority) does not need justification by the author -- it is totally reasonable to consider 89% an overwhelming majority.

(B) is incorrect because only one conclusion is drawn.

(C) is incorrect because the author does not overlook that possibility, but in fact admits it, saying "IF the rate is high."

(D) is incorrect. There is nothing contradictory about "wanting to finish" and "dropping out."

(E) is totally right, as you pointed out.

Does that clear that up?
I host free online workshop/Q&A sessions called Zen and the Art of LSAT. You can find upcoming dates here: http://www.manhattanlsat.com/zen-and-the-art.cfm
 
skapur777
Thanks Received: 6
Atticus Finch
Atticus Finch
 
Posts: 145
Joined: March 27th, 2011
 
 
 

Re: Q4 - A group of 1,000 students was randomly selected

by skapur777 Fri Apr 08, 2011 1:48 am

Oh, wow. I feel like an idiot. I feel like I shouldn't look too 'into' these questions because I make mistakes like that...but not looking into things in the LR section goes against the entire spirit of the LSAT!
User avatar
 
WaltGrace1983
Thanks Received: 207
Atticus Finch
Atticus Finch
 
Posts: 837
Joined: March 30th, 2013
 
 
trophy
Most Thanked
trophy
Most Thankful
trophy
First Responder
 

Re: Q4 - A group of 1,000 students

by WaltGrace1983 Sun Jan 26, 2014 3:16 pm

I thought that (D) was a fairly tempting answer but I think that the main thing to pay attention to in the conclusion is that word if. If you didn't see that word there, it would be incredibly easy to get (C) or (D) as an answer. However, (D) is wrong because there is nothing contradictory about saying "wanting" to do something and something actually happening.

-98% of Americans want a Lamborghini
-only 0.05% of Americans have a Lamborghini

Those are not contradictory at all.
 
mkd000
Thanks Received: 0
Jackie Chiles
Jackie Chiles
 
Posts: 38
Joined: March 14th, 2015
 
 
 

Re: Q4 - A group of 1,000 students

by mkd000 Wed Aug 05, 2015 10:58 pm

I got this question correct and understand it and all. I believe this question has a second flaw but I would like some feedback as I am not 100% sure. Is there a term shift going on as well? From "plan to finish high school" to "desire" (planning does not necessarily entail desire - maybe students are being encouraged to finish high school or else their parents will refuse to leave them trust funds).

Please let me know what you think! Thanks
 
mkd000
Thanks Received: 0
Jackie Chiles
Jackie Chiles
 
Posts: 38
Joined: March 14th, 2015
 
 
 

Re: Q4 - A group of 1,000 students

by mkd000 Thu Oct 08, 2015 9:46 pm

Was reviewing this and realized no one has got around to answering this. Please address! Thank you - your help is always much appreciated and valued.
User avatar
 
maryadkins
Thanks Received: 641
Atticus Finch
Atticus Finch
 
Posts: 1261
Joined: March 23rd, 2011
 
 
 

Re: Q4 - A group of 1,000 students

by maryadkins Mon Oct 12, 2015 12:05 pm

Indeed! :) Good find.